e-GMAT Logo
NEUR
N

If Q is an odd number and the median of Q consecutive integers is 120, what is the largest of...

GMAT Number Properties : (NP) Questions

Source: Official Guide
Number Properties
Statistics
MEDIUM
...
...
Notes
Post a Query

If \(\mathrm{Q}\) is an odd number and the median of \(\mathrm{Q}\) consecutive integers is \(120\), what is the largest of these integers?

A
\(\frac{\mathrm{Q} - 1}{2} + 120\)
B
\(\frac{\mathrm{Q}}{2} + 119\)
C
\(\frac{\mathrm{Q}}{2} + 120\)
D
\(\frac{\mathrm{Q} + 119}{2}\)
E
\(\frac{\mathrm{Q} + 120}{2}\)
Solution
  1. Translate the problem requirements: We have Q consecutive integers (where Q is odd), their median is 120, and we need to find the largest of these integers. "Consecutive integers" means integers that follow one after another (like 5, 6, 7, 8, 9), and "median" is the middle value when numbers are arranged in order.
  2. Visualize the symmetric arrangement: Since we have an odd number of consecutive integers, the median will be exactly in the middle, with an equal number of integers on both sides.
  3. Determine the position relationship: Calculate how many integers are to the right of the median to find the largest integer's position relative to the median.
  4. Apply the position offset: Add this offset to the median value (120) to find the largest integer.

Execution of Strategic Approach

1. Translate the problem requirements

Let's break down what we're given in everyday terms:

  • Q consecutive integers: This means we have Q numbers that follow one right after another, like 7, 8, 9, 10, 11 (that would be 5 consecutive integers)
  • Q is odd: This is important because it means we'll have a clear middle number
  • Median is 120: The median is the middle value when we arrange all numbers in order. Since Q is odd, the median will be exactly one of our integers
  • Find the largest: We need to determine what the biggest number in our sequence is

Since we have consecutive integers and Q is odd, the median (120) sits exactly in the center of our sequence.

Process Skill: TRANSLATE - Converting the problem language into clear mathematical understanding

2. Visualize the symmetric arrangement

Let's picture this with a concrete example first. If we had 5 consecutive integers with median 120:

118, 119, 120, 121, 122

Notice that 120 is exactly in the middle, with 2 numbers to its left and 2 numbers to its right.

For any odd number Q of consecutive integers with median 120, we get this pattern:

[some numbers] ... 120 ... [some numbers]

Since Q is odd, there's exactly one number in the middle (our median 120), which means there are equal numbers of integers on both sides.

Process Skill: VISUALIZE - Using concrete examples to understand the symmetric structure

3. Determine the position relationship

Now let's figure out how many numbers are to the right of 120 (our median).

If we have Q consecutive integers total, and one of them is the median (120), then:

  • We have Q - 1 remaining integers that are not the median
  • Since the arrangement is symmetric, half of these are to the left of 120, and half are to the right
  • So there are \(\frac{\mathrm{Q} - 1}{2}\) integers to the right of 120

Let's verify with our example: 5 consecutive integers means Q = 5

Numbers to the right of median = \(\frac{5 - 1}{2} = \frac{4}{2} = 2\)

This matches our sequence 118, 119, 120, 121, 122 where we have 2 numbers (121, 122) to the right of 120.

4. Apply the position offset

To find the largest integer, we need to move from the median (120) to the rightmost position.

Since there are \(\frac{\mathrm{Q} - 1}{2}\) integers to the right of 120, the largest integer is:

\(120 + \frac{\mathrm{Q} - 1}{2}\)

We can rewrite this as:

\(\frac{\mathrm{Q} - 1}{2} + 120\)

Let's verify with our example where Q = 5:

Largest = \(\frac{5 - 1}{2} + 120 = \frac{4}{2} + 120 = 2 + 120 = 122\)

Final Answer

The largest of the Q consecutive integers is \(\frac{\mathrm{Q} - 1}{2} + 120\)

Looking at our answer choices, this matches choice (A).

Answer: (A)

Common Faltering Points

Errors while devising the approach

1. Misunderstanding what "Q consecutive integers" means when Q is odd

Students often struggle to visualize that Q consecutive integers with an odd Q will have exactly one middle number (the median). They might incorrectly think the median falls between two numbers, leading them to set up the problem incorrectly from the start.

2. Forgetting to use the constraint that Q is odd

The problem explicitly states Q is odd, which is crucial for determining that 120 is exactly one of the integers (not an average of two middle numbers). Students might ignore this constraint and try to solve for even Q cases, leading to incorrect setup.

3. Misinterpreting the symmetric arrangement around the median

Students may not recognize that consecutive integers are symmetrically arranged around the median. They might try complex approaches involving algebraic sequences instead of using the simple fact that equal numbers of integers exist on both sides of the median.

Errors while executing the approach

1. Incorrect calculation of how many numbers are to the right of the median

Students might calculate this as \(\frac{\mathrm{Q}}{2}\) instead of \(\frac{\mathrm{Q} - 1}{2}\). They forget to exclude the median itself when counting the remaining numbers, leading to the wrong offset calculation.

2. Arithmetic errors when adding the offset to the median

Even with the correct offset of \(\frac{\mathrm{Q} - 1}{2}\), students may incorrectly write the final expression. They might write \(120 - \frac{\mathrm{Q} - 1}{2}\) instead of \(120 + \frac{\mathrm{Q} - 1}{2}\), confusing the direction of movement from median to largest.

3. Confusion between finding the largest versus smallest integer

Students might correctly identify that there are \(\frac{\mathrm{Q} - 1}{2}\) positions from the median but then move in the wrong direction, calculating \(120 - \frac{\mathrm{Q} - 1}{2}\) to find what they think is the largest integer.

Errors while selecting the answer

1. Choosing algebraically equivalent but differently written expressions

Students might arrive at the correct expression \(120 + \frac{\mathrm{Q} - 1}{2}\) but fail to recognize that this equals \(\frac{\mathrm{Q} - 1}{2} + 120\) in choice (A). They might incorrectly select choice (C) \(\frac{\mathrm{Q}}{2} + 120\) thinking it's the same thing.

2. Not verifying the answer with a concrete example

Students might select an answer without checking it against a simple case like Q=5. This verification step would catch errors like choosing (B) \(\frac{\mathrm{Q}}{2} + 119\), which would give \(\frac{5}{2} + 119 = 121.5\) for our example instead of the correct 122.

Alternate Solutions

Smart Numbers Approach

Step 1: Choose a convenient value for Q

Since Q must be odd, let's choose Q = 5 (a small odd number that makes calculations manageable).

Step 2: Set up the consecutive integers

With Q = 5 consecutive integers and median = 120, we need 5 consecutive integers where the middle one is 120.

Since there are 5 integers, the middle one is the 3rd integer.

So our integers are: 118, 119, 120, 121, 122

The largest integer is 122.

Step 3: Test this result against the answer choices

Using Q = 5 and checking each answer choice:

  1. \(\frac{\mathrm{Q} - 1}{2} + 120 = \frac{5 - 1}{2} + 120 = \frac{4}{2} + 120 = 2 + 120 = 122\)
  2. \(\frac{\mathrm{Q}}{2} + 119 = \frac{5}{2} + 119 = 2.5 + 119 = 121.5\)
  3. \(\frac{\mathrm{Q}}{2} + 120 = \frac{5}{2} + 120 = 2.5 + 120 = 122.5\)
  4. \(\frac{\mathrm{Q} + 119}{2} = \frac{5 + 119}{2} = \frac{124}{2} = 62\)
  5. \(\frac{\mathrm{Q} + 120}{2} = \frac{5 + 120}{2} = \frac{125}{2} = 62.5\)

Step 4: Verify with another value

Let's try Q = 7 to confirm our answer.

With 7 consecutive integers and median = 120: 117, 118, 119, 120, 121, 122, 123

The largest integer is 123.

Testing choice (A): \(\frac{7 - 1}{2} + 120 = \frac{6}{2} + 120 = 3 + 120 = 123\)

Conclusion: The smart numbers approach confirms that answer choice (A) is correct.

Answer Choices Explained
A
\(\frac{\mathrm{Q} - 1}{2} + 120\)
B
\(\frac{\mathrm{Q}}{2} + 119\)
C
\(\frac{\mathrm{Q}}{2} + 120\)
D
\(\frac{\mathrm{Q} + 119}{2}\)
E
\(\frac{\mathrm{Q} + 120}{2}\)
Rate this Solution
Tell us what you think about this solution
...
...
Forum Discussions
Start a new discussion
Post
Load More
Similar Questions
Finding similar questions...
Previous Attempts
Loading attempts...
Similar Questions
Finding similar questions...
Parallel Question Generator
Create AI-generated questions with similar patterns to master this question type.